Scope
I have a hard time seeing why the correct answer choice is right here because it seems to not dir...
shafieiava on May 17, 2020
  • December 2010 LSAT
  • SEC3
  • Q22
1
Reply
Why not choice D?
Struggling to see why choice D is not correct. Thank you!
rebut on May 15, 2020
  • December 2010 LSAT
  • SEC3
  • Q23
1
Reply
Answer choice D
I was wondering why D wouldn't be correct. If you infer the "something" and "it" as the backup sy...
NateT on April 3, 2020
  • December 2010 LSAT
  • SEC3
  • Q8
1
Reply
Answer choice c
Hi can you please Provide a more detailed explanation as to why C is incorrect? Thanks!
amf on March 16, 2020
  • December 2010 LSAT
  • SEC3
  • Q25
1
Reply
Why is E wrong?
Can you explain this to me?
nivensdc on March 12, 2020
  • December 2010 LSAT
  • SEC3
  • Q5
1
Reply
B
Can you please expound further on why B is wrong? From what I understand its B is wrong because w...
tomgbean on November 22, 2019
  • December 2010 LSAT
  • SEC3
  • Q21
3
Replies
Please diagram.
Please diagram - I am not sure why C is incorrect.
a42 on November 4, 2019
  • December 2010 LSAT
  • SEC3
  • Q21
2
Replies
It would not let me post this question/response...
I totally understand the diagramming for this passage; however, the "but not" in answer choice D ...
#JW on October 17, 2019
  • December 2010 LSAT
  • SEC3
  • Q19
1
Reply
Choice E
What's wrong with E?
Meredith on October 17, 2019
  • December 2010 LSAT
  • SEC3
  • Q14
3
Replies
Choice E
What's wrong with E?
Meredith on October 16, 2019
  • December 2010 LSAT
  • SEC3
  • Q5
1
Reply
general question
Can someone explain the actual question: "The statements above, if true, provide a basis for reje...
ash on September 16, 2019
  • December 2010 LSAT
  • SEC3
  • Q24
2
Replies
(B)
Hi LSAT Max, I chose answer choice (E) but was wondering why (B) can be safely eliminated sin...
Julie-V on August 30, 2019
  • December 2010 LSAT
  • SEC3
  • Q4
1
Reply
Whole-to-Part Breakdown
Hi LSAT Max, I see where the whole-to-part conflict is happening, but I would like some clari...
Julie-V on August 30, 2019
  • December 2010 LSAT
  • SEC3
  • Q7
1
Reply
Answer Explanation
Hi LSAT Max, When I first did this problem, it seemed as if none of the answer choices were a...
Julie-V on August 30, 2019
  • December 2010 LSAT
  • SEC3
  • Q2
1
Reply
Why is answer choice D incorrect?
Why is answer choice D incorrect? Is it because it merely restates a premise from the stimulus?
Samantha-Alexis on August 4, 2019
  • December 2010 LSAT
  • SEC3
  • Q20
4
Replies
I think part of the question stem is missing
I think part of the question stem is missing
dace on August 3, 2019
  • December 2010 LSAT
  • SEC3
  • Q6
3
Replies
Explanation
Can you please explain why E is the correct answer
missyargaw14 on February 26, 2019
  • December 2010 LSAT
  • SEC3
  • Q12
3
Replies
Help
I got this right, but I ran out of time for this question and picked from a c and e. Can someone ...
alymathieu on January 23, 2019
  • December 2010 LSAT
  • SEC3
  • Q9
1
Reply
Question Stem?
Hey Guys, The question stem for this question is not showing? Not sure sure if it's just my iPad...
Titan on December 20, 2018
  • December 2010 LSAT
  • SEC3
  • Q13
4
Replies
Need help!
Can someone please explain why C doesn't strengthen the argument and E does, thanks
Cynthia-Lee on September 17, 2018
  • December 2010 LSAT
  • SEC3
  • Q16
2
Replies